AP Human Geography Final Review

Pataasin ang iyong marka sa homework at exams ngayon gamit ang Quizwiz!

Although many factory laborers in MDCs are paid $________ per hour, factory laborers in LDCs may be paid less than $________ per hour. (A) 35; 2 (B) 75; 15 (C) 35; 10 (D) 100; 20 (E) 45; 2

(A) 35; 2

Which of the following is NOT true? (A) A family has migrated from the south side of Chicago to a town beyond the suburban ring because they like the schools and housing prices there. This is an example of counter urbanization (B) The three major kinds of push factors are political, economic, and environmental (C) The most common environmental threat to people comes from too much or too little water (D) Most migration occurs because of a combination of push and pull factors (E) Countries with slow population growth but strong economies commonly experience out-migration

(A) A family has migrated from the south side of Chicago to a town beyond the suburban ring because they like the schools and housing prices there. This is an example of counter urbanization

In the early twenty-first century, the largest number of refugees is located on which of the following continents? (A) Africa (B) Australia (C) Europe (D) North America (E) South America

(A) Africa

Which of the following is unique among Latin American countries in that its capital represents the concept of a forward capital? (A) Brazil (B) Argentina (C) Chile (D) Mexico (E) Peru

(A) Brazil

Which of the following is the best example of a footloose industry? (A) Call Centers (B) Steel (C) Furniture (D) Wine Making (E) Tuna

(A) Call Centers

The labor intensive clothing industry including the production of textiles (woven fabrics) and of apparel (clothing) is dominated by which paired countries? (A) China, India (B) Taiwan, South Korea (C) Mexico, Haiti (D) China, Singapore (E) Malaysia, Japan

(A) China, India

Bulk-gaining products include all of the following EXCEPT (A) Copper wire (B) Beverage bottling (C) fabricated metal (D) Motor vehicles (E) computers

(A) Copper wire

A Mormon church located in a rural area of northwestern Colorado is most likely the result of what kind of diffusion? (A) Expansion (B) Relocation (C) Maladaptive (D) Stimulus (E) Node Free

(A) Expansion

The Industrial Revolution began in ________ during the ________. (A) Great Britain, 1700s (B) Japan, 1800s (C) Russia, 1900s (D) the US, 1800s (E) Fertile Crescent, 1900s

(A) Great Britain, 1700s

Physiological population density is viewed as a superior measure of population density for which of the following reasons? (A) It is more reflective of population pressure on arable land (B) It yields the average population density (C) It more reflective of the world's largest population concentrations (D) It measures the average by dividing total land area by total number of people (E) It best reflects the percentage of a country's population that is urbanized

(A) It is more reflective of population pressure on arable land

This country is recognized as one of the most ethnically homogeneous in the world. (A) Japan (B) China (C) India (D) Bangladesh (E) Taiwan

(A) Japan

A taboo against pork is a characteristic of (A) Judaism and Islam (B) Judaism and Buddhism (C) Christianity and Buddhism (D) Christianity and Hinduism

(A) Judaism and Islam

Which of the following is an example of an ethnic religion and a universalizing religion with food taboos? (A) Judaism, Islam (B) Confucianism, Hinduism (C) Buddhism, Hinduism (D) Bahia, Christianity (E) Roman Catholicism, Islam

(A) Judaism, Islam

Which of the following is an example of a stateless nation or a nationality that wants to become a nation-state? (A) Kurdistan (B) Iceland (C) Poland (D) Denmark (E) Japan

(A) Kurdistan

Which of the following religions is predominant in the states of Wisconsin, Minnesota, and North Dakota? (A) Lutherans (B) Mormons (C) Baptists (D) Catholics (E) Methodists

(A) Lutherans

Which of the following is a correct statement about the major cities of the world? (A) Most are located on rivers or seacoasts. (B) Most are found in areas that are not very suitable for agriculture. (C) Most primate cities are located in the United States and Western Europe. (D) They are concentrated between the tropic of Cancer and the tropic of Capricorn. (E) The world's fastest-growing cities are found in areas with the highest standards of living.

(A) Most are located on rivers or seacoasts.

Which best describes the main difference between ethnicity and nationality? (A) Nationality is connected to a state while ethnicity is connected to culture (B) Nationality is connected to culture while ethnicity is connected to a state (C) Nationality is connected to a language while ethnicity is connected to a religion (D) Nationality is connected to cities while ethnicity is connected to rural areas (E) Nationality is connected to religion while ethnicity is connected to urban enclaves

(A) Nationality is connected to a state while ethnicity is connected to culture

On a Mercator projection map, where will you find the landmasses most exaggerated in relative size? (A) Near the poles (B) Actually, land masses aren't exaggerated on a Mercator map (C) Near the prime meridian (D) Near the equator (E) Near the major oceans

(A) Near the poles

In the United States, the largest concentrations of Amish, Latter-Day Saints, and Baptists are found in? (A) Pennsylvania, Utah, and South Carolina (B) Minnesota, Michigan, and Illinois (C) Ohio, Arkansas, and Kentucky (D) New York, Rhode Island, and West Virginia (E) Massachusetts, Idaho, and Texas

(A) Pennsylvania, Utah, and South Carolina

During the nineteenth and twentieth centuries, which of the following types of intraregional migration was the most prominent worldwide? (A) Rural to urban (B) Urban to suburban (C) Inland to coastal (D) Urban to rural (E) Suburban to urban

(A) Rural to urban

A growth boundary is associated with (A) Smart Growth Programs (B) utility networks (C) Urban Density Reduction programs (D) emergency medical services (E) Captive Growth Programs

(A) Smart Growth Programs

European imperial powers created _________ boundaries on much of Africa ignoring cultural and tribal differences across space. (A) Superimposed (B) Subsequent (C) Colonial (D) Antecedent (E) Territorial

(A) Superimposed

Why are most Latin American population centers located at or near the coast? (A) The colonial economies were export oriented (B) The wars of independence damaged many inland cities (C) Few Amerindian population centers were located on the coast (D) Latin American armed forces have extensive plans for defense of coastal sites (E) The climate is generally warmer on the coast and cooler in the highlands

(A) The colonial economies were export oriented

All of the following experienced sharp, sustained economic growth in the early 1990s EXCEPT (A) Vietnam (B) South Korea (C) Taiwan (D) Singapore (E) Hong Kong

(A) Vietnam

In the geopolitical sphere, a balance of power is (A) a condition of roughly equal strength between opposing sides, as if they were two equally matched camps (B) always bipolar but reinforced by satellite states. (C) a condition of unequal strength between opposing sides, as if they were two powerful allies, supported by a range of satellite states (D) exemplified by a League of Nations or, in the post-World War II era, the United Nations (E) the type of power shared by the members of the North Atlantic Treaty Organization

(A) a condition of roughly equal strength between opposing sides, as if they were two equally matched camps

The current distribution of soccer demonstrates that (A) a folk custom can become part of a popular culture (B) all sports are examples of folk culture (C) television has infused all sports into popular culture (D) American football is also an example of a folk culture

(A) a folk custom can become part of a popular culture

A nationality is (A) a group of people tied to a place through legal status and tradition (B) a country (C) an ethnic identity (D) any cohesive group of people (E) any group with shared religion, language, and origin of birth

(A) a group of people tied to a place through legal status and tradition

French influence on land division in the United States and Canada resulted in (A) a long-lot system (B) the metes and bound system (C) nucleated villages and fragmented farms (D) a rectilinear subdivision of public land (E) the township and range survey system

(A) a long-lot system

In comparing Malthus's theory to actual world food production and population growth during the past half-century, the principal difference is that (A) actual food production has been much higher than Malthus predicted (B) Malthus's theory predicted much higher food production than has actually occurred (C) actual population growth has been much higher than Malthus predicted (D) Malthus's theory predicted much higher population growth than has actually occurred

(A) actual food production has been much higher than Malthus predicted

In the United States many farms are integrated into a large food production industry. This is known as (A) agribusiness (B) commercial farming (C) food processing (D) mechanized farming (E) mixed crop and livestock farming

(A) agribusiness

British and American English differ in all but which of the following? (A) alphabet (B) pronunciation (C) spelling (D) vocabulary

(A) alphabet

Which pair of concepts or entities from South Africa and the United States is the best match? (A) apartheid - "Jim Crow" laws (B) homelands - blockbusting (C) South African Nationalist Party - U.S. Tea Party (D) Nelson Mandela - white flight (E) apartheid - U.S. Libertarian Party

(A) apartheid - "Jim Crow" laws

Pioneer farmers settling the grasslands of the American West often built houses of sod, while early settlers of the eastern forest built wooden structures like log cabins. This suggests that building materials (A) are strongly influenced by local resources (B) are commonly imported over long distances because of local folk culture (C) are chosen because of the diffusion of popular culture (D) are a uniform feature of folk culture (E) are elements of popular culture

(A) are strongly influenced by local resources

In the winter wheat area of the United States, the crop is planted in (A) autumn and harvested in summer (B) winter and harvested in spring (C) winter and harvested in autumn (D) spring and harvested in summer (E) spring and harvested in autumn

(A) autumn and harvested in summer.

The areas on the periphery of cities in less developed countries are sometimes known as (A) barriadas, favelas bidonvilles, bastees, or kampongs (B) young city and old city zones (C) public housing, barmiadelas, fonelongas, or kuhpinongs (D) the zone in transition, suburbs, or public zone (E) suburbs, barmiadelas, fonelongas, or kuhpinongs

(A) barriadas, favelas bidonvilles, bastees, or kampongs

A ________ is a large and fundamental division within a religion (A) branch (B) denomination (C) sect (D) church (E) puja

(A) branch

Los Angeles receives goods from Asia via ships. In the ports, the goods are put on trains for distribution around the US. Los Angeles is a(n) (A) break-of-bulk point (B) variable-trading partner (C) entrepot (D) backwash effect (E) footloose industry

(A) break-of-bulk point

In a simplified model of a city, the zone where retail and office activities are clustered is the (A) central business district (B) central commerce zone (C) urbanized downtown area (D) metropolitan statistical area (E) hub

(A) central business district

Hydroelectric power is the second most popular source of electricity after (A) coal (B) wind turbines (C) biomass (D) solar (E) nuclear power

(A) coal

Romance branch languages have achieved worldwide importance because (A) colonial and imperial activity distributed these languages to far-reaching territories (B) they are more frequently taught in middle schools today (C) they closely approximate English in grammar and vocabulary (D) they are the languages of governments (E) they alternate being the global lingua franca

(A) colonial and imperial activity distributed these languages to far-reaching territories

The underground CBD is characterized by (A) communication networks, subway trains, and pedestrian passages (B) high threshold retail services (C) public services such as municipal offices (D) high-end boutique shops serving downtown workers (E) chain restaurants

(A) communication networks, subway trains, and pedestrian passages

Which shape most easily fosters the establishment of effective internal communications for a smaller state? (A) compact (B) elongated (C) fragmented (D) Prorupted (E) prolonged

(A) compact

President Barack Obama is a good example of the (A) complexity of ethnic identity in the United States (B) confusion over ethnicity and race in Kenya (C) natural and biological basis for classifying humans (D) complexity of Asian American identity in the United States (E) principle of the distribution of persons of color

(A) complexity of ethnic identity in the United States

Diffusion of AIDS is an example of which type of diffusion? (A) contagious (B) hierarchical (C) relocation (D) stimulus

(A) contagious

A stimulus strategy is introduced by governments to (A) counter negative conditions of economic downturns (B) ensure TNC stock prices don't collapse (C) provide education funds in developed countries (D) provide health benefits to all citizens (E) promote research and development

(A) counter negative conditions of economic downturns

Farming varies around the world because of ________ across space. (A) cultural and environmental factors (B) cultural and economic factors (C) farmers' personal preference and environmental factors (D) weather and climate (E) climate change

(A) cultural and environmental factors

Correlating economic, social, and demographic indicators of development shows that (A) different indicators of development are associated with each other (B) a more developed country is likely to rank among the top ten in all major development indicators (C) a less developed country can see which indicators need improvement (D) economic, social, and demographic characteristics do not in reality coincide with each other

(A) different indicators of development are associated with each other

A(n) ________ is a place with high concentration of an ethnic group distinct from a surrounding area. (A) enclave (B) neighborhood (C) census block (D) county (E) urban area

(A) enclave

Compared to the private automobile, public transportation offers more (A) energy efficiency and less pollution (B) flexibility and energy efficiency (C) pollution and extensive transportation networks (D) privacy and less congestion of roads (E) range and threshold

(A) energy efficiency and less pollution

In ___________ religions, community, common history, and social relations are inextricably intertwined with spiritual beliefs. (A) ethnic (B) universal (C) monotheistic (D) evangelical (E) animism

(A) ethnic

A Syrian family migrated from Syria through Turkey to Greece without immigration documents in 2013. Although Greece does not grant the family refugee status and they face the risk of deportation, theirs is a case of international and ________ migration. (A) forced (B) human rights (C) voluntary (D) economic (E) transnational

(A) forced

Which was the most common religious change among Africans as a result of contact with people from Europe and the Middle East? (A) from animism to a universalizing religion (B) form an ethnic religion to animism (C) from a universalizing religion to an ethnic religion (D) from monotheism (E) from polytheism to an ethnic religion

(A) from animism to a universalizing religion

In contrast to popular culture, folk cultures are more likely to vary (A) from place to place at a given time (B) from time to time at a given place (C) both from place to place and from time to time (D) neither from place to place nor from time to time

(A) from place to place at a given time

An increasing number of states have adopted a federal form of government primarily to (A) grant different ethnicities or nationalities more effective representation (B) encourage the breakup of the superpower alliances (C) govern compact states more effectively (D) deploy scarce resources efficiently (E) accommodate rightwing political parties and their demands for more representation in national elections

(A) grant different ethnicities or nationalities more effective representation

British cities are surrounded by open space known as (A) greenbelts (B) public housing (C) sprawl (D) squatter settlements (E) suburbs

(A) greenbelts

The European Union has (A) has as its main objective to promote development within member states through economic and political cooperation (B) consolidates all economic planning in London, England (C) distributes large urban populations to undeveloped rural areas in order to distribute development and wealth (D) negotiates restrictive immigration policies and reduces mobility within member states (E) established English as the national language of all member states

(A) has as its main objective to promote development within member states through economic and political cooperation

A pidgin language (A) has no native speakers. (B) is spread by folk culture. (C) stems from invasion. (D) cannot exist for more than a generation. (E) cannot exist without globalization

(A) has no native speakers.

Mexico's maquiladora plants (A) have an advantage of proximity to U.S. markets (B) employ more than five million laborers (C) have an advantage of proximity to Latin American markets (D) must operate far from the U.S.-Mexico border (E) have grown in number as factories have been closed in China

(A) have an advantage of proximity to U.S. markets

Cultural ecology studies the (A) interactions between human societies and the physical environment (B) interactions between places over space (C) relationship between media and diffusion (D) relationship between language and land use (E) relationship between politics and policy

(A) interactions between human societies and the physical environment

Cultural ecology studies the (A) interactions between human societies and the physical environment (B) interactions between places over spaces (C) relationship between media and diffusion (D) relationship between language and land use (E) relationship between politics and policy

(A) interactions between human societies and the physical environment

In contrast to Fordist production, Post-Fordist production is more likely to (A) introduce more flexible work rules (B) place more importance on site factors (C) assign each worker one task (D) include repetitive tasks (E) require larger inventories of parts and components

(A) introduce more flexible work rules

Ethnicity is important because (A) it reinforces diversity in the face of the globalization of culture (B) it opposes nationalism and globalization (C) it promotes peace and wellbeing in the face of the globalization of culture (D) it provides the only stable basis of political states in the modern world (E) it defines citizenship and sovereignty in the political arena

(A) it reinforces diversity in the face of the globalization of culture

The strongest criticism of suburbs argued that historically (A) low-income people and minorities are unable to live in some areas because of the high cost of the housing, the unfriendliness (or discrimination) of established residents, and fears that property values would decline if minorities were allowed to buy property there (B) low-income people and minorities are able to live in some areas because of the low cost of the housing, the friendliness of established residents, and the myth that property values would decline if minorities were allowed to buy property there (C) legal devices, such as requiring several small houses to sit on a large lot of land amid several different apartments, prevented low-income families from living in many suburbs (D) low-income people and minorities are unable to live in some areas because of the high cost of the private schools there, the unfriendliness of African American and Hispanic minorities there, and the fear that property values would not change if other minorities were allowed to buy property there (E) they encouraged the buying and selling of too many automobiles

(A) low-income people and minorities are unable to live in some areas because of the high cost of the housing, the unfriendliness (or discrimination) of established residents, and fears that property values would decline if minorities were allowed to buy property there

Maquiladoras of northern Mexico are (A) manufacturing outsourcing plants (B) illegal migrant labor camps (C) border squatter settlements (D) organic agricultural cooperatives (E) commercial produce farms

(A) manufacturing outsourcing plants

The contemporary distribution of languages around the globe is largely a result of (A) migration (B) the Internet (C) Rosetta Stone sales (D) social media (E) high school requirements to learn a second language

(A) migration

Because the motor vehicle assembly industry is typically described as a bulk-gaining industry as well as a just-in-time delivery system, the most important factor when selecting a location for a new Toyota or Honda assembly plant in the United States might be (A) minimizing the cost of shipping finished vehicles to its customers (B) minimizing the cost of shipping inputs and outputs in relation to customers (C) maximizing delivery time to customers (D) the location of railroads (E) the location of waterways

(A) minimizing the cost of shipping finished vehicles to its customers

Which of the following is the most common form of commercial agriculture in continental Europe between France and Russia? (A) mixed crop and livestock farming (B) dairy farming (C) grain farming (D) livestock ranching (E) Mediterranean agriculture

(A) mixed crop and livestock farming

Analysis of South America indicates that in South America (A) nearly all of the people are Christian (B) the great majority of the people are non-religious (C) nearly all of the people practice folk religions (D) the population is almost equally divided between Christians and Muslims (E) about one- fourth of the people are Christian, one- fourth non-religious, and the remainder practice folks religions

(A) nearly all of the people are Christian

Urbanization can be analyzed by looking at the increase in the ________ of people living in cities. (A) number and percentage (B) number and homogeneity (C) percentage and density (D) density and heterogeneity (E) size and density

(A) number and percentage

A fundamental difference between folk culture and popular culture is that folk culture (A) often sets a minority group apart from a region's general population (B) is adaptive to change over time (C) tends to diffuse rapidly across geographic space (D) loses some of its distinctive characteristics as it crosses national boundaries (E) represents universal values

(A) often sets a minority group apart from a region's general population

Typically, popular culture (A) originates in a number of locations at the same time (B) reflects the characteristics of a distinctive physical environment (C) experiences frequent changes through time and space (D) is practiced by small homogeneous groups (E) is practiced by small heterogeneous groups that become large homogeneous groups

(A) originates in a number of locations at the same time

The new international division of labor reflects the growing importance of (A) outsourcing (B) vertical integration (C) access to raw materials (D) new infrastructure (E) just-in-time inventory management

(A) outsourcing

Compared to more developed countries, less developed countries have a higher percentage of workers in which sector of the economy? (A) primary (B) secondary (C) tertiary (D) quinary (E) quarternary

(A) primary

The most significant impact that Great Britain's enclosure movement made on the rural landscape was to (A) produce more of a dispersed rural settlement pattern (B) reinforce the traditional clustered rural settlement pattern (C) discourage urbanization (D) increase the rural population (E) improve transportation

(A) produce more of a dispersed rural settlement pattern

Some features of U.S. material culture such as gas stations, supermarkets, and motels (A) promote a uniform landscape (B) reflect the preservation of folk culture (C) provide diversity on the U.S. landscape (D) promote diffusion of folk culture (E) show high regional variation

(A) promote a uniform landscape

The Canadian Parti Québécois (A) promotes French-language cultural values and advocates sovereignty (B) promotes French as a vigorous language (C) promotes French as a standard language (D) promotes French-language cultural values and unification with France (E) promotes a stronger literary tradition of French

(A) promotes French-language cultural values and advocates sovereignty

The decline in the number of farmers in MDCs can best be described as a consequence of (A) push/pull economic factors, including the lack of opportunity in rural areas and higher-paying jobs in urban areas (B) urban sprawl, including the development of rings of suburbs and exurbs around major metropolitan areas, which required larger amounts of farm production (C) push/pull economic factors, including greater opportunities in rural areas and lower-paying jobs in urban areas (D) the increase of populations in urban areas and the reduced population in rural areas (E) the spread of disease and starvation in rural areas, which quickly reduced the populations of farmers

(A) push/pull economic factors, including the lack of opportunity in rural areas and higher-paying jobs in urban areas

The main difference between languages in the same family, branch, or group is how (A) recently in time the languages were once the same (B) closely the speakers of each language live to one other (C) they correspond to the diffusion of free markets across much of the world (D) similar the cultures of the speakers of each language are (E) they all emerged at the same point in history, according to the Bible

(A) recently in time the languages were once the same

The UNHCR (United Nations High Commission for Refugees) identified 3 groups forced to migrate for political reasons. Who are they? (A) refugees, IDPs, asylum seekers (B) refugees, political prisoners, those displaced by earthquakes and tsunamis (C) asylum seekers, guest workers, IDPs (D) IDPs, refugees, indigenous persons (E) refugees, political exiles, indigenous persons

(A) refugees, IDPs, asylum seekers

European CBDs are similar to those in North America because they both contain (A) retail and office activities (B) extensive residential areas (C) large numbers of skyscrapers (D) structures inherited from medieval times (E) ancient Roman structures

(A) retail and office activities

The Brown v. Board of Education court decision ruled that (A) separate schools for blacks and whites were unconstitutional (B) "white flight" was morally wrong and should be curtailed (C) discriminatory lending practices and restrictive covenants were unconstitutional (D) separate facilities for blacks and whites were acceptable so long as they were of the same quality (E) separate schools for blacks and whites were constitutional but separate drinking fountains were unconstitutional

(A) separate schools for blacks and whites were unconstitutional

The spatial clustering of high tech industry in Silicon Valley and the Research Triangle was a result of (A) skilled labor and venture capital (B) tax holidays and climate amenities (C) skilled labor and climate amenities (D) venture capital and tax holidays (E) the Pacific Rim economy and brain drain

(A) skilled labor and venture capital

This form of gerrymandering has been particularly attractive for creating districts inclined to elect ethnic minorities. (A) stacked vote (B) wasted vote (C) excess vote (D) gender biased (E) anti-immigration biased

(A) stacked vote

HDI is function of (A) standard of living, education, and health (B) standard of living, environmental health, and democracy (C) education, health, and democracy (D) fair trade, sustainability, and education (E) low debt, education, and health

(A) standard of living, education, and health

Brain drain is (A) the large-scale emigration of talented people (B) the process by which people are given reference for migration (C) people forced to migrate for political reasons (D) a cultural feature that hinders migration (E) a net decline in literacy

(A) the large-scale emigration of talented people

Most of the world's people live in (A) the less-developed countries (B) the northern hemisphere (C) the more-developed countries (D) China (E) urban areas in the developed world

(A) the less-developed countries

The Nomadic Warrior Theory and the Sedentary Farmer Theory have been developed to explain the common ancestry of (A) the non-Romance branches of Indo-European languages (B) Romance Languages (C) Sino-Tibetan languages (D) Afro-Asiatic languages (E) Austronesian languages

(A) the non-Romance branches of Indo-European languages

Which map would have the smallest scale and lowest resolution? (A) world (B) continent (C) state (D) city

(A) world

A megacity is defined as a metropolitan area with a population greater than (A) 1 million (B) 10 million (C) 25 million (D) 5 million (E) 50 million

(B) 10 million

If the scale of a map is 1:24,000, then 1 inch on the map represents ________ FEET on Earth's surface. (A) 1 (B) 2,000 (C) 2,400 (D) 24,000

(B) 2,000

Match each concept of ethnicity and political geography to its description. 1. Ethnic Cleansing 2. Genocide 3. Devolution 4. Sovereignty 5. Stateless Nation A) the systematic killing of ethnic group(s) by another ethnic group---Rwanda B) an independent state with boundaries and a government---United States C) an ethnicity which has a region chosen but not yet united----Basques D) the movement of power from a national to a regional government---Scotland E) a systematic plan using terror and violence to force an ethnic group to move (A) A-1, B-2, C-3, D-4, E-5 (B) A-2, B-4, C-5, D-3, E-1 (C) A-3, B-1, C-2, D-5, E-4 (D) A-4, B-5, C-1, D-2, E-3 (E) A-5, B-2, C-4, D-1, E-2

(B) A-2, B-4, C-5, D-3, E-1

_________ forces work to pull countries apart, while ____________ forces work to bind them together. (A) Centripetal....centrifugal (B) Centrifugal....centripetal (C) Communist....democratic (D) Capitalist....socialist (E) Anocracy...democracy

(B) Centrifugal....centripetal

Architects and planners from the _____________ strove to introduce beauty and impose order on chaotic industrial cities with extravagant monuments, parks, and blend in classical Roman architecture. (A) Industrial City movement (B) City Beautiful movement (C) classic movement (D) modernist tradition

(B) City Beautiful movement

Which of the following is NOT true? (A) Some countries have more than one official language (B) English is the official language of the United States (C) English is the official language of more than 50 countries across the globe (D) English is not the official language of the United Kingdom or Australia (E) The continent of Europe has more official languages than any other continent in the world

(B) English is the official language of the United States

Ranching is a good example of which type of agricultural system? (A) Intensive subsistence cultivation (B) Extensive commercial cultivation (C) labor-intensive agriculture (D) Capital-intensive agriculture

(B) Extensive commercial cultivation

Official languages in Switzerland include all but which of the following? (A) Italian (B) Flemish (C) Romansh (D) French (E) German

(B) Flemish

Which of the following democracies represents a unitary state in which power is concentrated in the hands of the central government? (A) India (B) France (C) South Africa (D) Russia (E) the United States

(B) France

The formula of the populations of New York City and Los Angeles divided by their absolute distance equals the amount of connectivity and accessibility they have. Which idea best illustrates their relationship using a non-spatial geographic model? (A) Population pyramids (B) Gravity model (C) Distance decay (D) Space-time compression (E) Agricultural density

(B) Gravity model

The "Four Economic Tigers" of East and Southeast Asia include (A) China, Indonesia, Japan, and Taiwan (B) Hong Kong, South Korea, Singapore, and Taiwan (C) Japan, Philippines, South Korea, and Vietnam (D) Hong Kong, Indonesia, Thailand, and Vietnam (E) China, Philippines, Singapore, and Thailand

(B) Hong Kong, South Korea, Singapore, and Taiwan

Which of the following arguments help explain why seventy five percent of those employed in Export Processing Zones, such as maquiladoras, are women? I. Women have better educational qualifications than men. II. Women are paid less than men. III. Many employers consider women to be more dexterous than men IV. Many employers consider women more likely to organize unions than men. (A) I and II only (B) II and III only (C) II and IV only (D) I, II, and III only (E) I, II, III, and IV

(B) II and III only

What is the main difference between population clusters in Asia and Europe? (A) In Asia, most people live in urban areas (B) In Europe, most people live in urban areas (C) In Asia, most people make their living in the secondary (manufacturing) sector of the economy (D) In Europe, most people make their living in the primary (outdoor) sector of the economy (E) In Asia, the tertiary (service) sector of the economy is stagnant

(B) In Europe, most people live in urban areas

Which of the following countries has the largest number of Sunni Muslims and the country with the largest number of Shiite Muslims? (A) Iran, Iraq (B) Indonesia, Iraq (C) Egypt, Turkey (D) United Kingdom, Jordan (E) Saudi Arabia, Iran

(B) Indonesia, Iraq

With respect to the North American Free Trade Agreement (NAFTA) between the United States, Canada, and Mexico, which of the following statements is not true? (A) It resulted in increased trade among member nations (B) It outlines an eventual supranational governing body for North America (C) It led to a net trade deficit for the United States and Canada with Mexico (D) It facilitated the movement of tertiary-sector industries from the United States to Mexico (E) It eventually may include other Latin American countries

(B) It outlines an eventual supranational governing body for North America

Which of the following best describes Alfred Weber's analysis of location decisions? (A) It seeks to evaluate the impact of a single factor of production on location (B) It seeks to minimize costs among multiple inputs of production (C) It is applicable only in postindustrial economies with predictable spatial patterns (D) It depends on the availability of a single node, link, and input of production (E) It is applicable only in industrial economics with predictable spatial patterns

(B) It seeks to minimize costs among multiple inputs of production

Which of the following is INCORRECT about OPEC, the Organization of Petroleum Exporting Countries? (A) Its original members were the 7 major Persian Gulf states (B) It was organized after 9/11/2001 to control oil prices (C) It was established in 1960 to control oil production, profits, and pricing (D) It has increased its membership to include nations in Africa, SE Asia, and South America (E) It placed the 1973 oil embargo on the U.S. for aiding Israel

(B) It was organized after 9/11/2001 to control oil prices

The laws enacted in 1573 that specifically outlined how colonial Spanish cities were to be constructed were called the (A) Bills of the Americas (B) Laws of the Indies (C) American Indian Laws (D) Colonial Center Laws (E) Laws of the Real Corona

(B) Laws of the Indies

Which of the following characteristics applies to more-developed countries? (A) Early stages of epidemiological transition (B) Long life expectancies (C) High total fertility rates (D) Rapid Population Growth (E) Early stages of the demographic transition

(B) Long life expectancies

Which of the following is not a primate city? (A) Mexico City, Mexico (B) New York, U S (C) London, England (D) Hanoi, Vietnam (E) Buenos Aires, Argentina

(B) New York, U S

In Immanuel Wallerstein's world systems analysis, he suggested that there is a core and a periphery of nations. Where is the world's core located in which hemisphere? (A) Southern (B) Northern (C) Western (D) Eastern (E) Pacific Rim countries

(B) Northern

Which of the following supranational organization has fostered the economic growth of nations in developing regions of Asia, Africa, Asia, and South America by forming a cartel controlling the production and pricing of oil. (A) European Union (EU) (B) Organization of Petroleum Exporting Countries (OPEC) (C) North Atlantic Treaty Organization (NATO) (D) North American Free Trade Agreement (NAFTA) (E) United Nations (UN)

(B) Organization of Petroleum Exporting Countries (OPEC)

Which of the following is a TRUE statement regarding space-time compression? (A) Places seem to all look the same (B) Places seem to be getting closer together (C) Places are increasingly concentrated on maintaining their histories (D) Places are making a greater effort to converge activities to save time (E) Places are getting closer together in space but not in time.

(B) Places seem to be getting closer together

The "separate but equal" doctrine in the United States was legally established by (A) individual states (B) Plessy v. Ferguson (C) Brown v. Board of Education (D) the Missouri Compromise (E) the fourteenth amendment to the Constitution

(B) Plessy v. Ferguson

Which of the following is TRUE? (A) Folk culture is more likely to vary from place to place at a given time, while popular culture is more likely to vary from time to time at a given place (B) Popular culture is more likely to vary from place to place at a given time, while folk culture is more likely to vary from time to time at a given place (C) Folk culture rarely varies from place to place at a given time and popular culture variations are limited primarily to large urban centers (D) Popular culture is a limiting effect on variations of folk culture (E) Folk culture is a limiting effect on the variations of popular culture

(B) Popular culture is more likely to vary from place to place at a given time, while folk culture is more likely to vary from time to time at a given place

According to the theory of environmental determinism, which of the following areas would have the most productive settlements? (A) Tropical regions (B) Temperate regions (C) Mountainous regions (D) Arctic regions (E) Arid regions

(B) Temperate regions

Von Thunen emphasized which of the following factors in his model of agricultural land use? (A) Labor cost (B) Transportation cost (C) Fertilizer cost (D) Machinery costs (E) Seasonal fluctuation in prices of farm products

(B) Transportation cost

Which of the following regions is currently experiencing the fastest population growth? (A) Northern Asia (B) Tropical Africa (C) Eastern Europe (D) Central Asia (E) Northeast United States

(B) Tropical Africa

Asian Americans are clustered in what area of the United States? (A) Southwest (B) West (C) Plains states (D) Northeast (E) Southeast

(B) West

FDI is defined as the investment of ________ in the economy of another country. (A) a foreign government (B) a foreign company (C) a non-government organization (D) the World Bank (E) the IMF

(B) a foreign company

A lingua franca is (A) an English word that has entered the French language (B) a language understood by people who have different native languages (C) an extinct language that has been revived (D) an official language in a region of the world different from where the language originated (E) a language used by French colonial administrations

(B) a language understood by people who have different native languages

A creolized language is (A) extinct (B) a mix of indigenous and colonial languages (C) an isolated language family (D) a possible prehistoric superfamily (E) a revived formerly extinct language

(B) a mix of indigenous and colonial languages

The Gender Inequality Index (GII) (A) compares the level of development of women in a country to the average development level of women in the world (B) compares the levels of indicators for females to those of males within a country (C) is composed of the same measures as the HDI but is applied only to women instead of the entire population (D) combines economic and political indicators of empowerment (E) cannot be used as an indicator of development in the United States

(B) compares the levels of indicators for females to those of males within a country

Which of the following is not an indicator of a country's level of development? (A) infant mortality rate (B) crude death rate (C) age structure (D) natural increase rate (E) total fertility rate

(B) crude death rate

The various cultural regions that compose the United States demonstrate how (A) various cultural regions share some values and yet remain distinct (B) culture hearths spread their values through diffusion (C) clear lines divide cultural regions distinctly (D) cultural boundaries match political boundaries (E) each cultural region is composed of a variety of cultural realms

(B) culture hearths spread their values through diffusion

A number of local congregations united in a single legal and administrative body is a (A) branch (B) denomination (C) sect (D) dialect (E) family

(B) denomination

We can deduce from the examples of popular music, fashionable clothing, and electronic applications around us that the rapid diffusion of popular culture (A) discourages people in different places to adopt different customs (B) depends on modern communication systems (C) is an example of relocation diffusion (D) conserves natural resources (E) always occurs at the expense of folk culture

(B) depends on modern communication systems

Diffusion of Internet service is following the earlier pattern of television, except that (A) the United States' share of world use is expanding (B) diffusion is much faster (C) diffusion is much slower (D) the expansion of service is faster in Africa than Asia (E) the initial use was in less developed countries

(B) diffusion is much faster

The GII combines which three measures to derive a composite score? (A) employment, school years completed, voting history (B) empowerment, employment, reproductive health (C) empowerment, reproductive health, school years completed (D) employment, reproductive health, literacy rate (E) literacy rate, empowerment, reproductive health

(B) empowerment, employment, reproductive health

According to the sector model, if family X has an income of $100,000 and family Y has an income of $40,000 but family Z has an income of $115,000 (A) families Y and Z are likely to live in the same sector of the city (B) families X and Z are likely to live in the same sector of the city (C) families X and Y are likely to live in the same sector of the city (D) families X, Y, and Z are likely to live in the same sector of the city (E) none of these families are likely to live in the same sector of the city

(B) families X and Z are likely to live in the same sector of the city

The process that includes subdividing a house from single-family owner occupancy to multiple occupancy is (A) blockbusting (B) filtering (C) gentrification (D) redlining (E) urban blight

(B) filtering

The eighteenth and nineteenth century slave trade represents ________ African migration while the twentieth century relaxation of U.S. immigration quotas resulted in ________ Latin American and Asia migration. (A) chain; hierarchical (B) forced; voluntary (C) voluntary; forced (D) chain; voluntary (E) forced; forced

(B) forced; voluntary

A feature of the physical environment commonly used to separate states includes all but which of the following? (A) deserts (B) geometry (C) mountains (D) lakes (E) rivers

(B) geometry

During the 1900s, the steel industry in the United States and since 1990 in China (A) declined in recent years owing to a surge in production in Japan (B) grew due to proximity to primary inputs such as iron ore and coal (C) declined due to demand for foreign cars (D) increased as a result of trade agreements (E) declined due to restrictions on coal production

(B) grew due to proximity to primary inputs such as iron ore and coal

People are more productive in more developed countries because they (A) work harder (B) have access to more technology (C) have a low amount of value added per person (D) are consistently better educated (E) understand their jobs better than workers in less developed countries

(B) have access to more technology

In Rostow's stages of development model, economic maturity is characterized by (A) high unemployment (B) higher number of tertiary jobs (C) gender empowerment (D) high per capita incomes (E) sustainability

(B) higher number of tertiary jobs

The cottage industry system involved manufacturing (A) of hand-made luxury goods (B) in the home (C) of steam engines and other cottage-sized machines (D) in small factories (E) cheeses and pastries

(B) in the home

The multiple nuclei model (A) involves four linked CBDs (B) includes nodes such as a port, a university, airport, and a park (C) includes a nucleus in the CBD which is connected to a nucleus in the suburbs (D) links a seaport, an airport, and a railway station. (E) disregards the use of nodes

(B) includes nodes such as a port, a university, airport, and a park

Swahili in East Africa and English in global commerce are examples of (A) pidgin languages (B) lingua franca (C) revived languages (D) creole language (E) official language

(B) lingua franca

Development analysts expect that a more developed country will have a (A) low GII and a low HDI (B) low GII and a high HDI (C) high GII and low HDI (D) high GII and a high HDL (E) low HDI

(B) low GII and a high HDI

U.S. inner cities face fiscal problems because (A) federal and state funds are available (B) low-income people are concentrated there (C) middle-class families are attracted there (D) redlining is no longer legal (E) blockbusting has strengthened ethnic neighborhoods

(B) low-income people are concentrated there

With the end of the Cold War, (A) Russia has become a nation-state (B) military alliances have become more important in Europe (C) nationalities have been discouraged from expressing their cultural identities (D) economic competition among regions has become more important than military competition

(B) military alliances have become more important in Europe

The ability to move either temporarily or permanently is best described by the term (A) migration (B) mobility (C) circulation (D) voluntary migration (E) nomadism

(B) mobility

A(n) ________ is a state that contains more than one ethnicity with traditions of self-determination and self-government. (A) multi-linguistic state like Nigeria (B) multinational state like Russia and the United States (C) anocracy like Russia (D) city-state like Babylon in ancient times (E) federal state like Germany

(B) multinational state like Russia and the United States

A ___________ is a group of people with a common political identity, and a ________ is a country with recognized boundaries. (A) territory....federalism (B) nationality.....state (C) nation....territory (D) territory....state (E) state....nation

(B) nationality.....state

Three types of disruptions can affect just-in-time delivery manufacturing: (A) labor unrest, elections, highway construction (B) natural hazards, traffic, labor unrest (C) natural hazards, warehouse sales, quality control (D) highway construction, labor unrest, invoice management programs (E) outsourcing, traffic, elections

(B) natural hazards, traffic, labor unrest

Saudi Arabia has successfully employed the international trade alternative primarily because of (A) consumer spending (B) petroleum reserves (C) regional cooperation (D) traditional social customs (E) its strategic distribution location

(B) petroleum reserves

Typical medieval European urban settlements were characterized by the (A) dispersal of palaces, churches, and other important buildings throughout the town (B) placement of buildings around a central market square (C) demolition of ancient walls surrounding the town (D) provision of parks and open space surrounding important churches (E) broad avenues in residential districts

(B) placement of buildings around a central market square

Which change most reduced fertility rates in less developed countries? (A) building hospitals and healthcare facilities (B) providing more education for girls (C) implementing pro-natalist policies (D) discouraging the use of birth control (E) promoting fundamentalist religious values

(B) providing more education for girls

With respect to the relationship between culture, religion, and the physical environment? (A) few religions derive meaningful events from the physical environment (B) religious ideas may be responsible for some of the changes people make in the physical environment (C) religion is no longer an important source of identification for a distinct cultural group (D) All of the above are true (E) None of the above are true

(B) religious ideas may be responsible for some of the changes people make in the physical environment

Democracies and autocracies differ in three essential ways; they are (A) longevity of regime, citizen participation, and adoption of free market economies (B) selection of leaders, citizen participation, and checks and balances (C) longevity of regime, selection of leaders, and checks and balances (D) selection of leaders, adoption of free market economies, checks and balances (E) longevity of regime, checks and balances, membership in the United Nations

(B) selection of leaders, citizen participation, and checks and balances

What one eats is revealing of one's (A) economic standing (B) social, religious, and ethnic memberships (C) occupation (D) social, educational level, and religious memberships (E) socioeconomic status and age group

(B) social, religious, and ethnic memberships

After maize, the most important crop in the U.S. mixed crop and livestock region is (A) wheat (B) soybeans (C) barley (D) fruits and vegetables (E) sugar beets

(B) soybeans

Edge cities emerged as a consequence of (A) highway construction (B) suburbanization (C) gentrification (D) remanufacturing (E) census classification

(B) suburbanization

As a result of high land costs, the American CBD is characterized by (A) less intensive land use (B) the construction of skyscrapers (C) suburban sprawl (D) a high threshold and range (E) the reuse of existing buildings

(B) the construction of skyscrapers

English has achieved unprecedented acceptance globally due to (A) the refusal to use English in colonial and imperial expansion (B) the diffusion of command economies across much of the world (C) its role as the common language of a global economy and culture (D) the rise of folk culture and the decline of popular culture (E) the diffusion of religion to industrialized nations

(B) the diffusion of command economies across much of the world

Higher social heterogeneity in urban settlements means that (A) the people you know socially are probably the same ones you see at work (B) the people are more alike than in rural settlements (C) people compete for limited space (D) there are many different kinds of people in cities (E) people play a specialized role in the urban economy

(B) the people are more alike than in rural settlements

Prime agricultural land in the United States is being replaced with (A) military bases (B) urban land uses (C) nature preserves (D) oil drilling (E) grazing land use

(B) urban land uses

Which is a current intraregional migration trend in the United States? (A) rural to urban (B) urban to suburban (C) metropolitan to nonmetropolitan (D) net emigration from the northeast (E) seasonal migration in winter months

(B) urban to suburban

Assuming a world population of 5,700,000,000 and an annual growth rate of 1.6 percent, how many people will be added to the world's population next year? (A) 912,000 (B) 9,120,000 (C) 91,200,000 (D) 912,000,000. (E) 9,120,000,000

(C) 91,200,000

The second most populous ethnic group in the United States is (A) Latinos/Hispanics (B) Asian Americans (C) African Americans (D) American Indians and Alaska Natives (E) Austral-Asians

(C) African Americans

Elongated states may suffer from poor internal communication and difficulty defending borders. Which of the following is NOT an elongated state? (A) Malawi (B) Gambia (C) Botswana (D) Chile (E) Italy

(C) Botswana

Who first coined the term geography? (A) Plato (B)Aristotle (C) Eratosthenes (D) Socrates (E) Anaximander

(C) Eratosthenes

Which of the following is NOT an indicator of global gender inequality? (A) Women on average have only two-thirds of the income of men in MDCs (B) Women have much lower incomes than men in LDCs (C) Female life expectancy is less than males in every country of the world (D) Female literacy is much lower than males in Sub-Saharan Africa (E) Women hold less than one-fourth of managerial jobs in LDCs where data are available

(C) Female life expectancy is less than males in every country of the world

Visible (Material) culture includes which three essential items? (A) Food, Shelter, Car (B) Food, Education, Car (C) Food, Shelter, Clothing (D) Clothing, Education, Car (E) Shelter, Car, Education

(C) Food, Shelter, Clothing

What is the difference between GPS and GIS? (A) GPS uses GIS data (B) GIS uses GPS data to determine location (C) GIS is the layering of data, whereas GPS is the gathering of data (D) GPS is the layering of data, whereas GIS is the gathering of data (E) GPS correlates with GIS, but GIS does not correlate with GPS

(C) GIS is the layering of data, whereas GPS is the gathering of data

Which of the following countries is NOT a nuclear power? (A) the United States (B) China (C) Germany (D) Russia (E) France

(C) Germany

The Icelandic language has changed less than any other Germanic language because of (A) Iceland's close contact with other people and activities (B) migration by German tribes (C) Iceland's relative isolation from other places (D) the extinction of the East Germanic group (E) continuous exchange with Norway and Sweden

(C) Iceland's relative isolation from other places

What reason best explains why the rate of natural increase is greater in less developed countries than in more developed countries? (A) In less developed countries, people have fewer children due to having an industrial society (B) In more developed countries, people have more children due to having an agricultural (C) In less developed countries, people have more children due to having an agricultural society (D) In more developed countries, people have fewer children because children are economic assets (E) In less developed countries, people have fewer children due to having a service based economy

(C) In less developed countries, people have more children due to having an agricultural society

Which is NOT true about the Law of the Sea? (A) It identifies territorial waters up to 12 nautical miles as under state control (B) It identifies international waters as having no single state control (C) It identifies internal shoreline waters as international to ensure free trade (D) It identifies the exclusive economic zone as a zone of state exploitation of resources such as fish and oil (E) It identifies a contiguous zone in which a state may enforce pollution controls and levy taxes for its use

(C) It identifies internal shoreline waters as international to ensure free trade

With respect to the North American Free Trade Agreement (NAFTA) between the United States, Canada, and Mexico, which of the following statements is NOT true? (A) It resulted in increased trade among member nations (B) It outlines an eventual supranational governing body for North America (C) It led to a net trade deficit for the United States with Canada and Mexico (D) It facilitated the movement of tertiary-sector industries from the United States to Mexico (E) It eventually may include other Latin American countries

(C) It led to a net trade deficit for the United States with Canada and Mexico

Nigeria's language diversity is dominated by which language family? (A) Khoisan (B) Nilo-Saharan (C) Niger-Congo (D) Afro-Asiatic (E) Austronesian

(C) Niger-Congo

The world's largest state is (A) China (B) Canada (C) Russia (D) Alaska (E) India

(C) Russia

Looking at an issue such as the distribution of cancer from a local to a national perspective in geography would be an example of using (A) Map scale (B) Differentiated analysis (C) Scale of analysis (D) Continuous synthesis (E) Regional analysis

(C) Scale of analysis

Looking at an issue such as the distribution of cancer from a local to a national perspective in geography would be an example of using (A) Map scale. (B) Differentiated analysis. (C) Scale of analysis. (D) Continuous synthesis (E) Regional analysis

(C) Scale of analysis

What was a distinctive feature of the world's superpowers between the 1940s and 1980s compared to other eras? (A) The United States and the Soviet Union were superpowers for the first time (B) The two superpowers were never involved in wars (C) The number of superpowers was much lower than in the past (D) The superpowers had satellites (E) The British were an equal in military power to both the United States and the Soviet Union

(C) The number of superpowers was much lower than in the past

The Sustainable Development Goals replaced what global effort? (A) structural adjustment programs (B) stimulus programs (C) UN Millennium Goals (D) Fair Trade movements (E) World Bank lending practices

(C) UN Millennium Goals

The first widespread use of the nation-state concept came in (A) Mesopotamia (B) the Roman Empire (C) Western Europe (D) the United States (E) Southeast Asia

(C) Western Europe

This European country was divided post-World War II into a Democratic Republic and a Federal Republic. (A) the United Kingdom (B) Czechoslovakia (C) Yugoslavia (D) Germany (E) Poland

(C) Yugoslavia

An isogloss is (A) a form of a language spoken in a local area (B) a collection of unique words (C) a boundary between language regions (D) a blending of two language families (E) a line that separates literary traditions

(C) a boundary between language regions

The Chernobyl incident in 1986 in the Soviet Union and the 2011 incident in Japan after an earthquake involved (A) a nuclear explosion high in the atmosphere (B) a nuclear missile that misfired and exploded (C) a nuclear power accident (D) a massive leak and fire at an oil exploration platform (E) an explosion and fire at a natural gas field

(C) a nuclear power accident

An urban center that is disproportionately larger than the second largest city in a country and that dominates the country's social, political, and economic activities can be best classified as (A) a megalopolis (B) a conurbation (C) a primate city (D) an edge city (E) an imperial city

(C) a primate city

The close proximity of maquiladora in Mexico and pharmacies like CVS and Walgreen's are example of (A) entrepots (B) deglomeration (C) agglomeration (D) situation factors (E) site factors

(C) agglomeration

Sub-Saharan Africa's fragile states' status may be attributed to all but which of the following? (A) high population growth and poor health (B) extreme poverty (C) alignment with the Soviet Union during the Cold War (D) ethnic tensions and ethnic cleansing (E) problematic shapes

(C) alignment with the Soviet Union during the Cold War

Local Native American and African religions that teach a belief in a natural world full of spiritual beings and supernatural powers are often referred to as (A) branches (B) shamanistic (C) animists (D) denominational (E) local religions

(C) animists

The 38th parallel between the Koreas is a good example of both a ____________ and a _______________ (A) line of demarcation....superimposed boundary (B) geometric boundary......relic boundary (C) antecedent boundary....demarcation line (D) relic boundary .....cultural boundary (E) antecedent boundary.....geometric boundary

(C) antecedent boundary....demarcation line

Map projections attempt to correct for errors in (A) transferability (B) area, distance, scale, and proportion (C) area, distance, shape, and direction (D) distance, proximity, and topology (E) distance, shape, and lines of latitude and longitude

(C) area, distance, shape, and direction

This is a sovereign state that comprises an urban area and the surrounding countryside. (A) rural-state (B) nation-state (C) city-state (D) primate-state (E) homogeneous state

(C) city-state

Which model of diffusion has Wal-Mart used in the United States to prevent distance decay principles? (A) clustered primarily in the Sunbelt region (B) dispersed throughout the region (C) clustered in a saturation principle (D) primarily in South and West

(C) clustered in a saturation principle

Compared to the United States, poor families in European cities are more likely to be (A) clustered in inner-city neighborhoods (B) dispersed throughout the city (C) clustered in suburbs (D) distributed uniformly in the city (E) living along major boulevards

(C) clustered in suburbs

The body of customary beliefs, social forms and material traits that constitute a group's distinct traditions would best describe a specific population's (A) folklore (B) political agenda (C) culture (D) education system. (E) religious organization

(C) culture

The fact that meat accounts for one-third of all protein intake in developed countries but only one-tenth in developing ones suggests (A) developing nations are vegetarians by choice (B) there is a cultural taboo against meat in developing nations (C) developed nations can afford more meat per capita (D) developing nations do not have sufficient grazing land (E) developed nations do not produce sufficient grains to replace meat consumption

(C) developed nations can afford more meat per capita

Prior to the Industrial Revolution, the distribution of industry in Europe was ________ and diffused from __________ to ______________. (A) clustered, north to south (B) concentrated, east to west (C) dispersed, west to east (D) randomly distributed, south to north (E) concentrated, west to east

(C) dispersed, west to east

In order to become a global industrial power, Japan had to overcome which of the following problems? (A) high labor costs (B) abundant energy sources (C) distance from consumers (D) weak consumer demand (E) proximity to China

(C) distance from consumers

India and China are the world's two most populous countries. While China has instituted a strict population policy, India (A) for cultural reasons, encourages women to continue to reproduce (B) does not endorse birth control because of the Catholic majority (C) encourages lower fertility through education and access to family planning (D) has a similar policy as China (E) because of their agricultural system, encourages reproduction

(C) encourages lower fertility through education and access to family planning

Chinese tradition, Judaism, Hinduism, and African traditional represent which type of religion? (A) unaffiliated (B) universalizing (C) ethnic (D) folk (E) syncretic

(C) ethnic

As they have more contact with popular culture, women in less developed countries are more likely to (A) bear more children (B) obtain food for the family (C) gain more opportunities outside the home (D) reduce the practice of prostitution

(C) gain more opportunities outside the home

1:24,000 is an example of what kind of scale? (A) bar line (B) fractional scale (C) graphic scale (D) written scale

(C) graphic scale

CBDs in Europe (A) follow the concentric ring model (B) contain only government services (C) have higher numbers of residents and more day-to-day consumer services (D) have a limited variety of land uses (E) only house educational facilities and government offices

(C) have higher numbers of residents and more day-to-day consumer services

Terrorist activities (A) are restricted to foreigners instigating violence in another country (B) are only associated with al-Qaeda (C) hold every citizen responsible and view them as culpable for the actions they oppose and therefore view citizens as legitimate targets (D) ended with the death of Osama bin Laden (E) have been reduced in frequency due to drone attacks

(C) hold every citizen responsible and view them as culpable for the actions they oppose and therefore view citizens as legitimate targets

According to the international trade approach to development, a country should identify all but which of its following assets? (A) abundant mineral resources (B) high quality manufactured goods (C) imports that it should limit (D) international consumer preferences (E) abundant agricultural products

(C) imports that it should limit

Like hip hop music, music from the border region between the southwestern United States and northern Mexico might be expected to (A) reflect almost exclusively on themes of migration, agriculture or violence (B) be an aspect of popular culture which threatens to overwhelm local folk cultures (C) incorporate local, national, and global themes as it demonstrates an interplay between globalization and localism (D) feature potentially offensive lyrics in order to guarantee its widespread transmission via radio and television (E) have diffused originally from one district of a large city

(C) incorporate local, national, and global themes as it demonstrates an interplay between globalization and localism

Quaternary economic activities are those that (A) extract natural resources from the environment (B) transform raw materials into finished products (C) involve the collection, processing, and manipulation of information (D) involve the exchange of goods and the provision of services (E) involve the production of fresh produce for urban markets

(C) involve the collection, processing, and manipulation of information

When people who speak a given language migrate to a different location and become isolated from other members of their group (A) their language usually shows very little change even over a long period of time, despite the appearance of a small number of changes typical of different dialects (B) they immediately develop a literary tradition (C) isolation usually results in the differentiation of one language into dialects, followed eventually by two distinct languages (D) they lose their linguistic abilities (E) groups form multiple dialects

(C) isolation usually results in the differentiation of one language into dialects, followed eventually by two distinct languages

One approach the World Trade Organization does NOT take to reduce barriers to trade is (A) it reduces or eliminates quotas on imports (B) it encourages subsidies on exports (C) it discourages subsidies on exports (D) it protects intellectual property rights (E) it reduces restrictions on movement of money

(C) it discourages subsidies on exports

Which factor of production is geographically fixed? (A) labor (B) capital (C) land (D) business acumen (E) communications

(C) land

Central Place Theory predicts larger settlements are (A) more numerous and closer together (B) more numerous and farther apart (C) less numerous and farther apart (D) less numerous and closer together (E) more numerous

(C) less numerous and farther apart

Grain raised in the United States is used today primarily (A) human food (B) a source of fuel (C) livestock feed (D) an export to foreign countries

(C) livestock feed

In Wallerstein's world systems model, the core nations of the world are (A) situated in the present-day Middle East (B) located in the prosperous southern continents (C) located mainly in Europe and North America (D) located in the former Soviet republics (E) located in the heartland, not the Rimland

(C) located mainly in Europe and North America

Wilbur Zelinsky's model of migration predicted (A) women are more likely to migrate than men (B) long migration distances are more likely than short (C) migration characteristics vary with the demographic transition (D) intraregional migration is more important than interregional migration in terms of political and economic resources (E) migrants move most frequently for economic reasons

(C) migration characteristics vary with the demographic transition

Situation costs are critical to a firm that wishes to (A) avoid skilled laborers (B) minimize production costs inside the plant (C) minimize transport costs (D) identify unique characteristics of a particular industry (E) utilize a new technology

(C) minimize transport costs

Hitler's nationalist/expansionist philosophies drew in part from (A) self-determination (B) sound historical evidence (C) organic geopolitical theory (D) rimland theory (E) heartland theory

(C) organic geopolitical theory

Which is not a way to compare states using political geography? (A) size (B) shape (C) organic theory (D) political organization (E) type of state

(C) organic theory

People in the United States are attracted to suburbs in part because suburbs are characterized by (A) heavy traffic (B) lower opportunity for home ownership (C) private land surrounding the house (D) row houses and apartments (E) closer proximity to cultural institutions

(C) private land surrounding the house

The United Nations in New York City and the European Union in Brussels, Belgium, are examples of ______________ service supranational organizations and are located in a(n) __________________ . (A) consumer, megacity (B) business, megalopolis (C) public, world city (D) consumer, entrepot

(C) public, world city

An adjustment made to the GNI to account for differences among countries in the cost of goods is a (A) tax exemption (B) subsidy (C) purchasing power parity (D) per capita index (E) global pricing indicator

(C) purchasing power parity

Economic development through international trade is an example of what type of diffusion? (A) contagious (B) hierarchical (C) relocation (D) stimulus

(C) relocation

Which is not a form of expansion diffusion? (A) contagious (B) hierarchical (C) relocation (D) stimulus

(C) relocation

Folk cultures are spread primarily by (A) contagious diffusion. (B) hierarchical diffusion. (C) relocation diffusion. (D) stimulus diffusion.

(C) relocation diffusion.

Which is the most dramatic change in the geographic distribution of African Americans in the United States? (A) rural to urban within the state (B) change to sharecropping (C) relocation to northern cities (D) movement out of inner-cities (E) relocation to coastal cities

(C) relocation to northern cities

According to the concentric zone model, a city develops in a series of (A) corridors (B) nodes (C) rings (D) sectors (E) quadrants

(C) rings

Retail activities which tend to concentrate in the CBD include those which have (A) no threshold (B) no range (C) services for office workers (D) a need for rapid transportation (E) a need for large amounts of horizontal space

(C) services for office workers

Core-periphery models are generally based on the idea that (A) all world regions are equally developed (B) levels of social and economic development are fairly uniform between core areas and peripheral areas (C) sharp spatial contrasts in social and economic development exist between economic heartlands and outlying subordinate areas (D) the growth and prosperity of core countries has not been achieved at the expense of peripheral countries. (E) the economic and social heart of a territory should be at its geographic center

(C) sharp spatial contrasts in social and economic development exist between economic heartlands and outlying subordinate areas

Containerization was developed to facilitate long-distance transport by ________ before transferring to trucks and trains. (A) truck (B) train (C) ship (D) airplane (E) rocket

(C) ship

Brazil's census classifies its population according to (A) country of origin (B) economic class (C) skin color (D) religion (E) political affiliation

(C) skin color

Most migrants to the United States during the early twentieth century came from which part of Europe? (A) central (B) north and west (C) south and east (D) south and west (E) north and east

(C) south and east

All of the following are examples of forced migration EXCEPT (A) the Trail of Tears in the early 19th century. (B) the Atlantic slave trade. (C) the California Gold Rush in the mid-nineteenth century (D) the Irish Potato Famine from 1846 to 1856 (E) the Japanese internment camps during World War II

(C) the California Gold Rush in the mid-nineteenth century

Social area analysis attempts to explain (A) the changing location of retail and office activities in North American cities (B) the development of squatter settlements in developing countries (C) the distribution of different types of people in an urban area (D) which of the three models of urban structure is the most accurate in the United States (E) regions ideal for social services

(C) the distribution of different types of people in an urban area

Cultural landscapes concept of Carl Sauer can be defined as (A) the types of art, music, dance, and theater (B) the ways that people in different cultures perceive the environment (C) the forms superimposed on the physical environment by the activities of humans (D) the diversity of distinctive cultures within a particular geographic area (E) a particular area within a geographic region dedicated to cultural activities

(C) the forms superimposed on the physical environment by the activities of humans

Development is best defined as (A) the process of improving the conditions of people by diffusing the American way of life (B) the process of improving the conditions of people by becoming a federal state (C) the process of improving the conditions of people through the diffusion of knowledge and technology (D) the process of improving the conditions of people by shifting the economic ideology from an entirely capitalist mode to a blended economy (E) the process of improving the conditions of people by improving diplomatic ties

(C) the process of improving the conditions of people through the diffusion of knowledge and technology

According to the Central Place Theory, the range and the threshold are defined as the (A) without a range, there are not enough customers to support the business (B) without threshold, the distance is too far for people to go to partake in the goods and services offered. (C) the range determines the maximum distance that people are willing to travel to buy or enjoy something, while the threshold is the minimum number of customers needed for the business to survive (D) the greater the range, the higher the cost of the item (E) the greater the threshold, the less the cost of the item.

(C) the range determines the maximum distance that people are willing to travel to buy or enjoy something, while the threshold is the minimum number of customers needed for the business to survive

In more developed countries, employment is increasing in (A) the primary sector (B) the secondary sector (C) the tertiary sector (D) all three sectors

(C) the tertiary sector

If the IHDI is less than the HDI, this indicates (A) there are lower levels of inequality in that country than there are in the world (B) the levels of inequality are equal to the global average (C) there are higher levels of inequality in that country than there are in the world (D) There is no comparison between the two indices as they measure different things (E) The HDI is really a function of the IHDI so this condition is impossible

(C) there are higher levels of inequality in that country than there are in the world

Geographers examine migration patterns because (A) they are developing policies to stop people movement (B) they believe migration impedes society's natural transition to stage 5 of the demographic transition model (C) they are seeking to explain the patterns of cultural diffusion including linguistics, ethnicity and religion (D) they are seeking to explain tectonic activity (E) geographers are rarely involved in studies such as this

(C) they are seeking to explain the patterns of cultural diffusion including linguistics, ethnicity and religion

The main effect of modern communications on social customs has been (A) to preserve folk cultures, by increasing awareness of their uniqueness (B) to stimulate the diffusion of folk cultures around the world (C) to increase the similarity of social customs in different locations (D) to have little effect on the diffusion of social customs

(C) to increase the similarity of social customs in different locations

Why did Israel build a "separation fence" around the West Bank? (A) to take and maintain control of the holy city of Jerusalem (B) to encircle the new Palestinian homeland created under a United Nations plan (C) to provide a buffer zone between the West Bank and Israel proper (D) to prevent a future invasion by its Muslim neighbors (E) to deter Palestinian terrorist attacks on Israeli settlements in the region

(C) to provide a buffer zone between the West Bank and Israel proper

The number of functions in a central place is dependent on all of the following EXCEPT the (A) population of the central place (B) population of the market place (C) total number of central places in the urban system (D) distance to a place of similar function (E) size of the market place

(C) total number of central places in the urban system

Gross domestic product (GDP) is the (A) total value of the outputs minus inputs of goods and services produced in a country during a year (B) total value of exports of goods and services produced in a country during a year (C) total value of outputs of goods and services produced in a country during a year (D) total value of human capital development in a country during a year (E) total value of investments made in domestic and foreign sources of revenue during a year

(C) total value of outputs of goods and services produced in a country during a year

The Industrial Revolution (A) was a rapid introduction of modernizing technology across Europe and North America concurrently (B) was a rapid introduction of modernizing technology in Britain's colonies (C) was a gradual diffusion of new ideas in industry and social, economic and political inventions (D) introduced migration away from cities (E) started in North America before reaching Europe

(C) was a gradual diffusion of new ideas in industry and social, economic and political inventions

The principal difference between a multiethnic and multinational state is (A) multiethnic states speak more languages than multinational states (B) multinational states speak more languages than multiethnic states (C) whether different groups in the state desire for self-determination (D) the migration history of that state and its percentage of foreign-born people (E) the way the state's government is divided

(C) whether different groups in the state desire for self-determination

A legal form of segregation in U.S. cities is achieved through (A) blockbusting (B) redlining (C) zoning (D) greenbelts (E) busing

(C) zoning

According to the rank-size rule, if the largest city in a country has a population of 10 million, the next largest will have a population of (A) 9 million (B) 8 million (C) 7.5 million (D) 5 million (E) 3.5 million

(D) 5 million

Population in census tracts rarely exceeds (A) 500 (B) 1,000 (C) 2,500 (D) 5,000 (E) 50,000

(D) 5,000

Which of the following is NOT part of the definition of state? (A) A common language (B) An organized economy and circulation system (C) Political sovereignty (D) A permanent resident population (E) Defined boundaries

(D) A permanent resident population

Which of the following is a landlocked country? (A) Peru (B) Germany (C) Burma (D) Afghanistan (E) Colombia

(D) Afghanistan

Which country has created special economic zones (SEZs) to attract investment? (A) Japan (B) Vietnam (C) South Korea (D) China (E) Thailand

(D) China

Which of the following is NOT true regarding interregional migration? (A) Canadian interregional movement shifted west to east given its large Pacific Rim community (B) Soviet Russia's interregional migration was largely orchestrated by government policies to develop economic centers near resource rich areas (C) Brazilian interior development was orchestrated by the planned city development of Brasilia and flexible land use policies in the Amazon (D) China's rural to urban movement, up until recently, had been restricted through government policies (E) Russia's current interregional migration patterns are representative of less government mandated relocation and economic policies

(D) China's rural to urban movement, up until recently, had been restricted through government policies

Rostow's modernization model is concerned with which of the following concepts? (A) Dependency (B) Structuralism (C) Core-periphery relations (D) Economic development (E) Neocolonialism

(D) Economic development

The Dust Bowl in the Great Plains is the best example of which push or pull factor? (A) Political pull factor (B) Economic push factor (C) Environmental pull factor (D) Environmental push factor (E) Economic pull factor

(D) Environmental push factor

Which of the following is NOT true about acid precipitation? (A) It kills the flora and fauna in lakes (B) It injures plants by depriving them of nutrients on land (C) It corrodes marble and limestone buildings (D) It converts CFCs to acid (E) It has had the highest concentrations in the eastern states of the United States

(D) It converts CFCs to acid

Most migrants to the United States during the peak of the late nineteenth century came from which parts of Europe? (A) Balkan Peninsula (B) Scandinavian Peninsula (C) Great Britain (D) Italy and Greece (E) European Russia

(D) Italy and Greece

The world's four largest religions in terms of the total number of adherents include all but which of the following? (A) Christianity (B) Buddhism (C) Hinduism (D) Judaism (E) Islam

(D) Judaism

The Spine and Disamenity zones are most closely associated with a(n) (A) Apartheid city (B) Southeast Asian city (C) French Colonial city (D) Latin American city (E) Muslim city

(D) Latin American city

Today, most of the United States and Canadian (or MDCs) population lives in which of the following? (A) Farming areas (B) Rural non-farming areas (C) Central cities (D) Metropolitan areas (E) Small towns

(D) Metropolitan areas

Which of the following is NOT one of Ravenstein's migration laws? (A) Most migration is rural to urban (B) Migrants traveling long distances will likely settle in a big city (C) People in rural areas are more migratory than city dwellers (D) Most international migrants are young women (E) Most migration is step migration

(D) Most international migrants are young women

Which of the following is NOT true concerning recycling? (A) Aluminum scrap is readily accepted for recycling (B) Glass can be used without loss of quality (C) Plastics are symbolized with rankings for safety in recycling (D) Newspaper recycling has declined as paper pulp prices have declined (E) Mixed color glass is not profitable for recycling

(D) Newspaper recycling has declined as paper pulp prices have declined

Which U.S. state was judged by the Washington Post to have the most gerrymandering? (A) North Dakota (B) South Dakota (C) West Virginia (D) North Carolina (E) South Carolina

(D) North Carolina

The principal religion of Central and South America is (A) Eastern Orthodoxy (B) Islam (C) Buddhism (D) Roman Catholicism (E) Protestantism

(D) Roman Catholicism

Swaziland makes ________ into a perforated state. (A) Madagascar (B) Mozambique (C) South Sudan (D) South Africa (E) Zimbabwe

(D) South Africa

Which of the following includes the world's earliest centers of plant domestication? (A) British Isles, Scandinavia, United States (B) Northeast Asia, Eastern Europe, South Africa (C) Australia, New Zealand, China (D) Southeast Asia, Mesoamerica, Middle East (E) Russia, China, Latin America

(D) Southeast Asia, Mesoamerica, Middle East

According to the Anatolian Hearth model, seed agriculture probably reached Europe from (A) western India. (B) northern China. (C) Ethiopia. (D) Southwest Asia

(D) Southwest Asia

The model that illustrates both space and time being shortened through technology is called (A) Population pyramids (B) Gravity model (C) Distance decay (D) Space-time compression (E) Population density

(D) Space-time compression

Which of the following is an INCORRECT statement about the Cold War? (A) The United States and the USSR were the only two remaining superpowers after WWII (B) The United States reinforced its political alliances and ideology through military assertion in Panama and Grenada (C) Both the United States and the USSR established a military presence via bases and troop presence in countries who were allies (D) The Cuban Missile Crisis was negotiated and settled peacefully at the United Nations (E) Countries in the Soviet Bloc were described as satellites

(D) The Cuban Missile Crisis was negotiated and settled peacefully at the United Nations

Oil price history shows that prices have changed sharply on several occasions. Which is NOT an event that led to a sharp fluctuation in barrel price? (A) OPEC refused to sell oil to allies of Israel in 1973 (B) The end of the Iran-Iraq war in 1988 led to lower prices (C) Oil companies control supply of oil in Southwest Asia in the early 70s keeping prices low (D) The U.S. government removes all subsidies in 1980 to major oil companies like Exxon-Mobil causing an increased prices (E) The Iranian Revolution in 1979 and Iran-Iraq war trigger oil shortages and higher prices

(D) The U.S. government removes all subsidies in 1980 to major oil companies like Exxon-Mobil causing an increased prices

Which of the following is NOT true about the world's energy supply? (A) 33 percent of the world's natural gas is produced in Southwest Asia and Russia (B) Approximately 50 percent of the world's coal reserves are found in China (C) Russia and Saudi Arabia provide at least 25 percent of the world's petroleum (D) The United States produced 50 percent of the world's petroleum output (E) 33 percent of the world's natural gas comes from the United States and other developed countries

(D) The United States produced 50 percent of the world's petroleum output

Many Islamic states in the Middle East region combine religion and state and don't separate them. This is an example of what? (A) Monotheism (B) Plural society (C) Democracy (D) Theocracy (E) Multilingual society

(D) Theocracy

Many Islamic states in the Middle East region combine religion and state and don't separate them. This is an example of what? (A) Monotheism (B) Plural society (C) Democracy (D) Theocracy (E) Multilingual society

(D) Theocracy

Which of the following is NOT true about contemporary factories? (A) They tend to cluster in industrial parks (B) They tend to be located proximate to suburban highway junctions (C) They are best functioning as a single-story building (D) They require proximity to rail lines (E) They tend to locate on rural or suburban land because of lower taxes

(D) They require proximity to rail lines

The language spoken by soldiers stationed throughout the Roman Empire was known as (A) Official Latin (B) Romance language (C) Standard language (D) Vulgar Latin (E) Catalan Latin

(D) Vulgar Latin

According to the sector model, the best housing is located in (A) renovated inner-city neighborhoods (B) an outer ring surrounding the city (C) nodes near universities and parks (D) a corridor from downtown to the edge of the city (E) gated communities

(D) a corridor from downtown to the edge of the city

The transfer of money by migrants to family or community members in their country of origin is (A) a visa tax (B) a form of global aid (C) reimbursement for travel (D) a remittance (E) required to return home

(D) a remittance

Which generalization is not a part of Buddhist teachings? (A) the Four Noble Truths (B) Buddhists monks separate from society to meditate (C) no missionaries sent out to proselytize (D) all Buddhists can serve as monks (E) following the Eightfold Path

(D) all Buddhists can serve as monks

The South is established as a vernacular region of the United States by (A) climate (B) the Baptist Church (C) low high school graduation rates (D) all the above

(D) all the above

In contrast to the international trade approach, the self-sufficiency approach to development (A) begins when an elite group initiates innovative activities (B) attempts to result in uneven resource development. and market correction indices (C) suffers from market stagnation (D) attempts to spread investment through all sectors of the economy (E) attempts to identify appropriate developmental stages

(D) attempts to spread investment through all sectors of the economy

The Gravity Model illustrates spatial distribution of a service EXCEPT for (A) Customer call centers (B) shopping patterns (C) e-mails (D) automotive superstore (E) international trade

(D) automotive superstore

The choice of clothing in Western countries is strongly influenced (A) by occupation but not level of income (B) by level of income but not occupation (C) by knowledge of fashion elsewhere, as well as the level of folk culture (D) by occupation, income, and knowledge of fashion elsewhere (E) by fashion only

(D) by occupation, income, and knowledge of fashion elsewhere

The distribution of alcohol beverage choices in the United States displays which characteristic of popular culture? (A) rapid diffusion (B) the lack of a correlation to level of income (C) barriers owing to the physical environment (D) choice based on preferences for what is produced, grown or imported locally (E) uniform distribution across the landscape

(D) choice based on preferences for what is produced, grown or imported locally

In recent decades, all of the following have played a major role in the rapid growth on Sun Belt cities of the United States EXCEPT (A) immigration levels from Latin America (B) high levels of per capita federal spending in the South and West (C) cheap land and labor (D) climatic change leading to colder northern winters (E) the increasing demand for retirement and resort centers

(D) climatic change leading to colder northern winters

The predominant form of agriculture in the U.S. Southeast is (A) mixed crop and livestock (B) dairy farming (C) Mediterranean agriculture (D) commercial gardening (E) plantation farming

(D) commercial gardening

The secondary sector of the economy includes which of the following? (A) data processing (B) mining (C) government (D) construction (E) banking

(D) construction

Mixing crops and livestock allows farmers to (A) distribute the workload of the crops and livestock so that farmers hardly need to work during the summer season (B) generate 90 percent of their income from the sale of livestock (C) double crop (D) create a system where crops provide food for livestock and livestock provide manure for crop fertilization (E) circumvent market forces that determine the prices of livestock and crops

(D) create a system where crops provide food for livestock and livestock provide manure for crop fertilization

According to the gravity model, technological improvements in transportation and communications technology should (A) not affect the probability of interaction between two places (B) increase the population of the two places (C) decreases the amount of interaction between two places (D) decrease the friction of distance (E) decrease the population in the two places

(D) decrease the friction of distance

When a country experiences food insecurity, it (A) has eliminated the availability of periodic markets (B) has introduced policies that prevent restaurants from operating in a free market (C) is engaged in military conflict (D) does not have physical, social, and economic access to safe and nutritious food (E) does not have access to grocery stores

(D) does not have physical, social, and economic access to safe and nutritious food

Which pattern is more typical of folk cultures than other types of cultures? (A) welcoming in new practices from other cultures (B) promoting a high level of religious diversity (C) taking steps to transform rural areas into urban areas (D) emphasizing the value of tradition (E) establishing flexible gender roles

(D) emphasizing the value of tradition

U.S. quota laws from the 1920s until the 1960s had the effect of (A) virtually ending immigration (B) indirectly causing two world wars (C) dramatically increasing immigration from around the world (D) ensuring the majority of migrants continued to be from Europe (E) increasing the possibility of migration from regions previously prohibited

(D) ensuring the majority of migrants continued to be from Europe

The pull factor that was a significant factor for African Americans to move south to north in the early to mid-twentieth century was (A) inexpensive housing (B) equality under the law (C) education opportunities (D) factory jobs with higher wages (E) inexpensive agriculture land

(D) factory jobs with higher wages

The diffusion of social media has threatened (A) popular culture and folk culture relatively equally (B) the folk customs of only a few isolated communities (C) the spread of the international banking system (D) folk culture more than popular culture (E) folk culture, but only in the largest cities on each continent

(D) folk culture more than popular culture

Over the past 60 years, the number of sovereign state memberships in the United Nations (A) has remained approximately the same (B) has increased by 30 countries (C) has decreased by 30 countries (D) has increased by more than 50 countries (E) has increased by more than 100 countries

(D) has increased by more than 50 countries

Race is often described as (A) being characterized by Caucasian, African American, and Hispanic/Latino (B) evenly distributed around the world, independent of ethnicity (C) determinable from physical characteristics such as the exact shape of a person's face or head (D) identification with a group that is perceived to share a physiologic trait such as skin color (E) defined by statute in most U.S. states.

(D) identification with a group that is perceived to share a physiologic trait such as skin color

Which of the following examples in INCORRECT about the changing distribution of steel production at a global scale? (A) US was the leading producer in 1980 (B) China is leading producer as of 2013 (C) steel mini-mills are located near markets (D) integrated steel mills locate near outputs (E) scrap metal is key to industry

(D) integrated steel mills locate near outputs

Gerrymandering (A) is only done by Democrats to ensure control of the House (B) is only done by Republicans to ensure Congress is ineffective (C) is a Republican process to ensure control by the wealthy elite (D) is the redrawing of legislative boundaries by both parties (E) is the redrawing of legislative boundaries to ensure federal funds are distributed to those who need them the most

(D) is the redrawing of legislative boundaries by both parties

Development prospects are limited in Sub-Saharan Africa because of all but which of the following? (A) colonial legacy (B) poor leadership (C) capacity of land to produce food (D) lack of monsoon rains

(D) lack of monsoon rains

Regionalization would best explain the characteristics of the study of (A) popular culture (B) adaptive strategies (C) built environment (D) material culture (E) folk culture

(D) material culture

In the United States, which of the following definitions of a city covers the largest functional area? (A) central business district (B) central city (C) urbanized area (D) metropolitan statistical area (E) regional government federation

(D) metropolitan statistical area

The Brazilian government encouraged interregional migration by (A) making Rio de Janeiro and Sao Paulo more attractive (B) dictating optimal locations for factories (C) clearing the rainforest for agricultural activities in the interior (D) moving the capital to Brasilia (E) industrial development on the Atlantic coast

(D) moving the capital to Brasilia

Which of the following events would be considered a migration pull factor? (A) revolutionary takeover of a government (B) failed harvest (C) flooding of a river (D) opening of a new factory (E) civil war

(D) opening of a new factory

In less developed countries, consumer goods such as telephones, televisions, and motor vehicles are (A) available for sharing by a large number of people (B) unknown and unfamiliar to most people (C) essential to rural life (D) owned by a growing minority of the people (E) considered only luxuries for the wealthy

(D) owned by a growing minority of the people

Which indicator has shown a widening gap since 1980? (A) HDI (B) life expectancy (C) mean years of education (D) per capita GNI (E) All four progress indicators have shown a narrowing gap

(D) per capita GNI

The key elements of nationalism include all but (A) common culture (B) shared attitudes (C) shared songs (D) political structure (E) state symbols

(D) political structure

The Mall including the Capitol building and the Washington Monument in Washington, D.C. and the orderly street plan and expansive parks of Chicago, Illinois, are a good example of a(n) ___________ . (A) medieval landscape (B) symbolic landscape (C) military landscape (D) postmodern landscape

(D) postmodern landscape

If a geographer performs a study on people's perceptions of the Deep South using interviews as the primary data source, the geographer's method is (A) quantitative (B) systematic (C) anthropogenic (D) qualitative (E) idiographic

(D) qualitative

Gated communities best represent (A) a zoning ordinance (B) redlining (C) gerrymandering (D) segregation based on social class (E) a country club

(D) segregation based on social class

Inner city population densities were generally ________, speaking to the quality of life in inner cities. (A) comparable to suburban densities (B) equal to that found in rural areas (C) comparable to population density of the United States as a whole (D) significantly higher (E) impossible to measure because no census statistics were collected for inner cities

(D) significantly higher

Under communism, Eastern Europe was characterized by all but which of the following? (A) investment in heavy industries such as iron and steel (B) governments that made the key decisions concerning the national economy (C) favorable balances between population and resources (D) specialization in production of consumer-oriented products

(D) specialization in production of consumer-oriented products

Factories have reduced demand for petroleum by (A) consuming more coal (B) consuming more nuclear energy (C) switching to solar power (D) switching to natural gas (E) There has been an increase in consumption not a decrease

(D) switching to natural gas

The opening of the U.S. interior in the early 1800s was facilitated by (A) the completion of the transcontinental railway (B) the pull factor of the California Gold Rush (C) the pull factor of hydroelectric projects and controlled irrigation programs (D) the Erie Canal and the Northwest Ordinance (E) government mandated relocation during the War of 1812

(D) the Erie Canal and the Northwest Ordinance

A racist believes in (A) the equality of women and men regardless of ethnic or racial identity (B) the superiority of some groups because of cultural identity (C) the inferiority of some groups because of economic factors and the superiority of other groups because of political affiliations (D) the biological classification of people and the superiority of some groups over others on the basis of racial identity (E) the biological classification of people along with an understanding that all human beings are one species and therefore one extended family without any inherent differences

(D) the biological classification of people and the superiority of some groups over others on the basis of racial identity

What was apartheid? (A) the dialect of Dutch which is spoken in South Africa (B) South Africa's governmental system (C) the existence of landlocked states in southern Africa (D) the geographic separation of races in South Africa (E) the kinship system of Sub-Saharan Africa

(D) the geographic separation of races in South Africa

The geographic study of the distribution of languages provides a good example of (A) the diffusion of free markets across much of the world (B) the diffusion of folk culture in different areas of the world (C) the role and spread of religion across much of the world (D) the interplay between globalization and local diversity (E) political conflicts that arise due to ethnic tensions

(D) the interplay between globalization and local diversity

When the models of urban structure developed in Chicago are applied to São Paulo, one conclusion is that (A) both cities are located near large lakes (B) São Paulo doesn't have high income neighborhoods (C) the models don't work in São Paulo (D) the poorest people are located in different areas (E) physical geography has not influenced the distribution of social classes in São Paulo

(D) the poorest people are located in different areas

A literary tradition is (A) a form of a language intended to be printed in official government documents (B) specific only to Shakespearian English (C) a collection of languages related to one other (D) the written form of a language (E) the variety of dialects in a language used in obscure examples of poetry

(D) the written form of a language

Judaism, Christianity, and Islam are alike in that they all (A) began as universalizing religions (B) are examples of ethnic religions (C) evolved into polytheistic religions (D) trace their heritage to Abraham (E) started in a region that is now a part of Europe

(D) trace their heritage to Abraham

Which of the following have aided commercial farmers in MDCs? (A) transportation improvements and the rise of sea levels (B) scientific advances and the reduction in the need for electronics (C) ancient irrigation projects that can be refitted for modern farming (D) transportation improvements, scientific advances, and electronics (E) climate change, scientific advances, and transportation improvements

(D) transportation improvements, scientific advances, and electronics

The type of agriculture practiced near large cities, which includes producing fruits and vegetables, is called (A) sawah (B) truck agriculture (C) subsistence agriculture (D) truck farming (E) truck hybridization

(D) truck farming

In Antarctica, geometric boundaries do little to organize a vast (A) frontier (B) borderland (C) wasteland (D) tundra (E) territory

(D) tundra

The biggest problem in promoting development through the international trade alternative is (A) increased demand for many goods (B) increased price of petroleum (C) regional cooperation (D) unequal distribution of resources (E) consumer demand expanding faster than manufacturing can

(D) unequal distribution of resources

Where are Lutherans clustered in the ________ United States. (A) northeastern (B) southern (C) southwestern (D) upper Mid-western (E) northwestern

(D) upper Mid-western

Women played a crucial role in the domestication of plants because they (A) were interested in varying the diets of their families (B) knew how to achieve control over their environments (C) were agile climbers on the hillsides of the Fertile Crescent (D) were engaged in collecting plant resources.

(D) were engaged in collecting plant resources.

The U.S. center of population has moved steadily to the (A) east (B) north (C) south (D) west (E) center

(D) west

Carrying capacity is a function of (A) technology (B) natural resources (C) resource allocation (D) limiting factors (E) (A), (B), and (C)

(E) (A), (B), and (C)

During what 20-year period has the United States seen the greatest immigration in terms of raw numbers? (A) 1850 - 1870 (B) 1900 - 1920 (C) 1920 - 1940 (D) 1960-1980 (E) 1980-2000

(E) 1980-2000

Which of the following is an example of a cultural landscape? (A) Coastal wetland (B) Cloud forest (C) Stand of mangrove trees (D) Eroded shoreline (E) Adobe ruins

(E) Adobe ruins

Cultural landscape is closest in meaning to which of the following? (A) Cultural ecology (B) Nonmaterial culture (C) Environmental determinism (D) Physical geography (E) Built environment

(E) Built environment

Which of the following is useful for describing a settlement node whose primary function is to provide support for the population in its hinterland? (A) Von Thunen's model of land use (B) Concentric zone model (C) Core-periphery model (D) Rostow's model of economic development (E) Christaller's model of central place

(E) Christaller's model of central place

Which of the following activities is most likely to be found in the outermost zone of Von Thunen's model of agricultural land use? (A) Mixed farming (B) Subsistence farming (C) Specialty farming (D) dairying and market gardening (E) Extensive grain or livestock raising

(E) Extensive grain or livestock raising

Which of the following was NOT one of the five original urban hearth regions? (A) The Indus Valley (B) Mesopotamia (C) Mexico (D) North China (E) France

(E) France

Which of the following is the best definition of remote sensing? (A) Gathering information about ocean depth from radar (B) Gathering information about the earth's surface from GPS systems (C) Gathering information about the earth's surface from microwaves in space (D) Gathering information about the earth from geographical information systems (GIS) (E) Gathering information about the earth from great distance over broad areas

(E) Gathering information about the earth from great distance over broad areas

Place the following religious leaders in the correct order. I. Jesus of Nazareth II. Siddhartha Gautama III. Mohammed IV. Joseph Smith (A) I, II, III, IV (B) I, III, IV, II (C) IV, I, II, III (D) III, IV, II, I (E) II, I, III, IV

(E) II, I, III, IV

Which of the following correctly sequences the continuum from language family to dialect? (A) Afro-Asiatic, Semitic, Arabic, Berber (B) Sino-Tibetan, Sinitic, Mandarin, Chinese (C) Indo-European, Indo-Iranian, Hindi, Bengali (D) Indo-European, Balto-Slavic, Russian, Ukrainian (E) Indo-European, Germanic, English, Midland-Northern

(E) Indo-European, Germanic, English, Midland-Northern

The UN, when established after World War II, replaced an earlier global cooperation effort. What was it? (A) NATO (B) Warsaw Pact (C) Alliance for Peace (D) Organization of American States (E) League of Nations

(E) League of Nations

The more developed regions include all but which of the following? (A) North America (B) Eastern Europe (C) South Pacific (D) Western Europe (E) Middle East

(E) Middle East

Which of the following is NOT true? (A) Unaccompanied minors arriving from Central America to the United States are pushed because of gang-related activity (B) Movement between developing countries includes higher numbers of elderly migrants than between developed countries (C) Female immigrants include the group reuniting with husbands (D) Increased Mexican female migrations reflects the changing role of women in Mexico (E) Most of the immigrants to the United States are females between the ages of 40 to 65

(E) Most of the immigrants to the United States are females between the ages of 40 to 65

Which statement would best fit the Gravity model in relation to migration? (A) New York and Los Angeles are closely linked by distance (B) New York and Los Angeles are not linked at all by their distance (C) New York and Los Angeles are linked due to their proximity to ports (D) New York and Los Angeles are not linked due to their populations (E) New York and Los Angeles are linked due to their populations

(E) New York and Los Angeles are linked due to their populations

These two countries are committed to unifying the nation but face disagreement as to which will be the dominant political and economic ideology. (A) North and South Vietnam (B) Moldova and Romania (C) Lesotho and the Republic of South Africa (D) Morocco and Sahrawi Republic (E) North and South Korea

(E) North and South Korea

The two primary Eastern religions of Hinduism and Buddhism each have their hearths located where? (A) Jerusalem, Israel (B) Western Saudi Arabia (C) Eastern China (D) Southeast Asia (E) Northern India

(E) Northern India

What geographical approach suggests that humans possess the ability to dominate their environment rather than that they are defined by the environment? (A) Animistic approach (B) Temperate approach (C) Environmental determinist approach (D) Socioecological approach (E) Possibilism approach

(E) Possibilism approach

Which of the following is NOT a characteristic of the Fair Trade movement? (A) Standards intending to protect workers are instituted in LDCs (B) Fair Trade business practices increase entrepreneurial and managerial skills of producers (C) Cooperatives intend to benefit local farmers and artisans, rather than absentee corporate owners (D) Employers must pay fair wages and comply with environmental and safety standards (E) Protection of workers' rights is already a high priority for multinational corporations

(E) Protection of workers' rights is already a high priority for multinational corporations

Canada's northern region would be classified as what according to the core-periphery model? (A) Industrial Core (B) Upward transition (C) Downward transition (D) Semi-periphery (E) Resource frontier

(E) Resource frontier

Which world-city is the best example of an entrepot? (A) Shanghai (B) Chicago (C) Sao Paulo (D) Madrid (E) Singapore

(E) Singapore

Multiple nuclei models are closely associated with contemporary cities in which geographical region? (A) Southwest Asia (B) North Africa (C) Central Asia (D) Sub Sahara Africa (E) Southeast Asia

(E) Southeast Asia

Which of the following is NOT true? (A) The United Nations allows all member states to discuss conflicts rather than engage in warfare to solve problems (B) The United Nations provides an international team of peacekeeping troops to missions around the world (C) The United Nations plays a role in addressing global economic problems (D) The United Nations provides humanitarian aid and assistance (E) The United Nations experienced no substantial growth in membership since the 1960s when the colonial powers vacated African nations

(E) The United Nations experienced no substantial growth in membership since the 1960s when the colonial powers vacated African nations

Which of the following does NOT represent one of the United States' 3 main eras of immigration? (A) involuntary migration from Sub-Saharan Africa in the seventeenth and eighteenth centuries (B) voluntary migration from the British Isles due to religious persecution in the seventeenth and eighteenth centuries (C) Ellis Island became a major immigration processing center in the late nineteenth and early twentieth centuries (D) Early twentieth century immigrants from southern and eastern Europe coincided with the diffusion of the Industrial Revolution through continental Europe (E) The leading sources of twenty-first century Asian immigration are the result of climate change in places such as Bangladesh, Oceania, or the Maldives

(E) The leading sources of twenty-first century Asian immigration are the result of climate change in places such as Bangladesh, Oceania, or the Maldives

Which of the following refers to the size and functional complexity of cities? (A) Multiplier Effect (B) The Sector Model (C) Basic-non-basic industries (D) Threshold ratio (E) Urban hierarchy

(E) Urban hierarchy

According to its United Nations membership, this is the smallest European microstate yet one of the richest regions in the world? (A) Andorra (B) Monaco (C) Singapore (D) Malaysia (E) Vatican City

(E) Vatican City

Despite the risks to the environment, the U.S. Gulf Coast has become an important industrial area because of (A) shrimp trawling and seafood processing (B) just-in-time inventory management (C) proximity to markets in Southwest Asia (D) access to oil imports from Mexico and Venezuela (E) access to oil and natural gas fields

(E) access to oil and natural gas fields

Geographers define overpopulation as (A) too many people in the world (B) too many people compared to resources (C) too many people in a region (D) too many people in a country (E) all of the above

(E) all of the above

The Fertile Crescent (A) followed the Tigris and Euphrates rivers. (B) extended from the Persian Gulf to the Mediterranean Sea. (C) is another name for Mesopotamia. (D) is the focus of W.D. Pattison's sustainability studies. (E) all of the above

(E) all of the above

Dramatic increases in global grain production since 1950 have been made possible by (A) substantial increases in the amount of land under cultivation. (B) global warming (C) an increase in the urban workforce (D) an increase in the agricultural workforce (E) an increase in the use of energy and technology

(E) an increase in the use of energy and technology

Immigrants to the United States (A) are typically under the age of 15 (B) are typically over the age to 65 (C) are males between the ages of 16 and 20 (D) are females between the ages of 16 and 20 (E) are typically between the ages of 20 and 39

(E) are typically between the ages of 20 and 39

The principal benefit of the self-sufficiency approach is to promote (A) international trade (B) global competitiveness for local industries (C) the maintenance of a large bureaucracy (D) unequal distribution of resources (E) balanced growth of all economic sectors

(E) balanced growth of all economic sectors

Twenty-five U.S. states have so-called "right-to-work" laws that (A) are more numerous and anti-union in northern states than in southern states (B) send a powerful signal to employers that workers have the right to join a labor union, whether or not this unionization might cause inconveniences for management (C) make it much more difficult for employers to limit wages, cut health benefits, and stop unions from forming (D) fight the efforts of factory management to break unions and keep future unions from forming (E) cause a great deal of trouble for labor unions attempting to organize workers and bargain with employers

(E) cause a great deal of trouble for labor unions attempting to organize workers and bargain with employers

Most of the conflict in Africa is widespread because of (A) colonial boundaries clearly demarcating the various ethnic and national populations (B) numerous ethnic groups living in perpetual peace and understanding (C) rapid economic development for the poor at the expense of the rich (D) gradual economic development favoring the poor over the rich (E) colonial boundaries in the midst of numerous ethnic and national groups

(E) colonial boundaries in the midst of numerous ethnic and national groups

State support for terrorism is seen to include all but one of the following. (A) providing sanctuary for wanted terrorists (B) assisting in planning terrorist attacks (C) supplying arms and materiel for terrorist attacks (D) supplying both money and intelligence to terrorist groups (E) establishing a political party representing terrorist grievances

(E) establishing a political party representing terrorist grievances

An examination of the distribution of ethnicities in the United States reveals (A) ethnicities are not often clustered in urban areas (B) different ethnicities cluster in each U.S. region (C) ethnic neighborhoods contain a heterogeneous mix of ethnicities, even in cities that were once known for their patterns of segregation (D) segregation and exclusion are a thing of the past for nearly all U.S. ethnic groups (E) ethnic groups tend to cluster in urban areas and in different U.S. regions

(E) ethnic groups tend to cluster in urban areas and in different U.S. regions

According to the cultural complex described in the diagram, auto ownership provides transportation but also (A) represents a set of American cultural traits, such as self-reliance and independence (B) provides greater likelihood that its owner will attain American values such as wealth (C) represents one American cultural trait, depending on the value's of the auto owner (D) comes about as a result of possession of a particular American trait, such as wealth (E) helps to identify the owner's cultural landscape, region, realm, and level of freedom

(E) helps to identify the owner's cultural landscape, region, realm, and level of freedom

The Nile River Valley has the greatest concentration of population in Egypt because of its (A) alpine climate (B) tourist attractions (C) extensive in-migration from nearby areas (D) many sacred sites (E) high-quality agricultural land

(E) high-quality agricultural land

All of the following describe the English language EXCEPT (A) it is a lingua franca (B) its recent growth is due to expansion diffusion (C) it is an Indo-European language (D) it has diffused along with economic globalization (E) it uses logograms

(E) it uses logograms

Fracking is most closely associated with extracting (A) coal (B) copper (C) diamonds (D) molybdenum (E) natural gas

(E) natural gas

According to Homer Hoyt's sector model, once a district with high-class housing is established, the most expensive new housing is built (A) on the outer edge of the suburban area, farther out from the center (B) on the inner edge of that district, closer to the center (C) in the skyscrapers of the CBD (D) in old industrial buildings and retail shops (E) on the outer edge of that district, farther out from the center

(E) on the outer edge of that district, farther out from the center

The Germans established the ________ known as the Caprivi Strip in present-day Namibia to access resources in central Africa, including the Zambezi River. (A) causeway (B) disruption zone (C) railroad (D) protraction (E) proruption

(E) proruption

Which of the five means of transportation would fruit production in South America for market in the US need to use? (A) Inland waterways (B) Overland trucks (C) Airplanes (D) Rail transport (E) refrigerated ship

(E) refrigerated ship

The dependency ratio is most useful for indicating the (A) reliance of a country on imported fossil fuels (B) degree of gender equality within a country (C) relationship between the total fertility rate and the infant mortality rate (D) percentage of foreign ownership within the secondary sector of a country's economy (E) relationship between the potential labor force and the remainder of a country's population

(E) relationship between the potential labor force and the remainder of a country's population

The most prominent type of intraregional migration in the world is (A) north to south (B) region to region (C) urban to rural (D) city to city (E) rural to urban

(E) rural to urban

The primary sector of the economy includes which of the following? (A) data processing (B) beverage bottling (C) government (D) banking (E) sheep herding

(E) sheep herding

What two location factors influence more industries to remain in the northeastern United States and central Europe? (A) global communications and fluid capital (B) cheap labor and high consumer demand (C) availability of raw materials and cheap energy (D) fluid capital and advantageous tariffs (E) skilled labor and rapid delivery to market

(E) skilled labor and rapid delivery to market

Accessibility and connectivity are two interrelated ways to describe (A) Absolute location (B) Spatial concentrations (C) Relative directions (D) geographical sites (E) spatial interactions

(E) spatial interactions

Austerity programs (A) target cuts in private industry payroll (B) introduce furlough days in private firms (C) introduce new taxes on water and electricity (D) target the dissolution of unions (E) target cuts in government spending on welfare benefits

(E) target cuts in government spending on welfare benefits

The greatest total number of foreign-born residents can be found in (A) China (B) Australia (C) Germany (D) the United Kingdom (E) the United States

(E) the United States

Franglais, Denglish, and Spanglish represent (A) the corruption of grammar in European languages (B) lingua francas intended to compete with official languages (C) Vulgar Latin (D) scheduled languages (E) the infusion of English into French, German and Spanish and subsequent development of new words

(E) the infusion of English into French, German and Spanish and subsequent development of new words

Which is an example of sequent occupance? (A) the forced movement of Native Americans from their homelands to reservations (B) the Diaspora of the Jews after being banished from their homeland by the Romans (C) the decision by Amish immigrants to settle in rural areas rather than in urban enclaves. (D) an ethnic group that is completely surrounded by another ethnic group (E) the shift in a neighborhood population from Germans to Poles to Koreans

(E) the shift in a neighborhood population from Germans to Poles to Koreans

In the nineteenth and early twentieth century, the demographic transition in Europe was best characterized by (A) a shift in the composition of national populations toward greater ethnic balance (B) a net population decline resulting from an excess of deaths over births (C) migration between Europe countries (D) large-scale population movements following periods of war or widespread civil unrest (E) urbanization and falling birth rates

(E) urbanization and falling birth rates

Basque is a good example of a (A) vigorous language family (B) vigorous and globalizing language (C) vigorous language group (D) vigorous lingua franca (E) vigorous and isolated language

(E) vigorous and isolated language

The boundary between the United States and Canada is best described by which of the following? (A) geometric only (B) linguistic and religious (C) water and linguistic (D) mountain and water (E) water and geometric

(E) water and geometric

Official languages become institutionalized when they A) are used by the government, in education, by business, and by the mass media B) ensure that everyone in the country speaks that language C) require that all books and publications be produced in that language only D) require all media to broadcast in that language E) are spoken only in international and diplomatic negotiations

A) are used by the government, in education, by business, and by the mass media

Which of the following best demonstrates the concept of a culture hearth (A) the Middle East, where many cultures share the religion of Islam (B) Latin America, which consists of several distinct but related cultures (C) the places in the world where material and nonmaterial traits emerged (D) the buildings, roads, and other elements built by humans (E) an area within a city where people share a common culture distinct from the surrounding culture

C) the places in the world where material and nonmaterial traits emerged


Kaugnay na mga set ng pag-aaral

Human Communication final exam Chap. 7,8,10,11

View Set

Money and Banking Final Exam Multiple Choice

View Set

BUSINESS 101 CHAPTER 8:Producing Quality Goods & Services

View Set

Biology of IR (Ligand Receptor Pairs involved in T cell Activation)

View Set

Layers of the Atmosphere Quizlet

View Set

AP Computer Science A- Primitive Types

View Set

MISSED Practice Exam Questions: Life and Health

View Set

Praxis 5206 Teaching Reading K-12

View Set